Difference between revisions of "1999 IMO Problems/Problem 2"
(problem and somewhat unattractive smoothing solution) |
m (→Solution: typo fix) |
||
Line 12: | Line 12: | ||
== Solution == | == Solution == | ||
− | The answer is <math>C=1/8</math>, and equality holds exactly when two of the <math>x_i</math> are equal to each other and all the other <math>x_i</math> are zero. We prove this by induction on the number of nonzero <math>x_i</math>. | + | The answer is <math>C=1/8</math>, and equality holds exactly when two of the <math>x_i</math> are equal to each other and all the other <math>x_i</math> are zero. We prove this by [[induction]] on the number of nonzero <math>x_i</math>. |
− | First, suppose that at most two of the <math>x_i</math>, say <math>x_a</math> and <math>x_b</math>, are nonzero. Then the left-hand side of the desired inequality becomes <math>x_a x_b (x_a^2 + x_b^2)</math> and the right-hand side becomes <math>(x_a + x_b)^4/8</math>. By AM-GM, | + | First, suppose that at most two of the <math>x_i</math>, say <math>x_a</math> and <math>x_b</math>, are nonzero. Then the left-hand side of the desired inequality becomes <math>x_a x_b (x_a^2 + x_b^2)</math> and the right-hand side becomes <math>(x_a + x_b)^4/8</math>. By [[AM-GM]], |
<cmath> \begin{align*} | <cmath> \begin{align*} | ||
x_a x_b(x_a^2 + x_b^2) = 1/2 \left[ \sqrt{(2x_ax_b)(x_a^2 + x_b^2)} \right]^2 | x_a x_b(x_a^2 + x_b^2) = 1/2 \left[ \sqrt{(2x_ax_b)(x_a^2 + x_b^2)} \right]^2 | ||
Line 22: | Line 22: | ||
with equality exactly when <math>x_a = x_b</math>, as desired. | with equality exactly when <math>x_a = x_b</math>, as desired. | ||
− | Now, suppose that our statement holds when at most <math>k</math> of the <math>x_i</math> are equal to zero. Suppose now that <math>k+1</math> of the <math>x_i</math> are equal to zero, for <math>k\ge 2</math>. Without loss of generality, let these be <math>x_1 \ge \dotsb \ge x_k \ge x_{k+1}</math>. | + | Now, suppose that our statement holds when at most <math>k</math> of the <math>x_i</math> are equal to zero. Suppose now that <math>k+1</math> of the <math>x_i</math> are equal to zero, for <math>k\ge 2</math>. [[Without loss of generality]], let these be <math>x_1 \ge \dotsb \ge x_k \ge x_{k+1}</math>. |
We define | We define | ||
<cmath> y_j = \begin{cases} x_j, & j \notin \{k, k+1\} \\ | <cmath> y_j = \begin{cases} x_j, & j \notin \{k, k+1\} \\ | ||
Line 42: | Line 42: | ||
<cmath> S(x_k + x_{k+1})^3 = S(x_k + x_{k+1})^3 + 3S(x_k^2 x_{k+1} + x_k x_{k+1}^2), </cmath> | <cmath> S(x_k + x_{k+1})^3 = S(x_k + x_{k+1})^3 + 3S(x_k^2 x_{k+1} + x_k x_{k+1}^2), </cmath> | ||
but none of the other terms change. Since <math>3S > S \ge a_1 \ge a_k, a_{k+1}</math>, it follows that we have strictly increased the right-hand side of the equation, i.e., | but none of the other terms change. Since <math>3S > S \ge a_1 \ge a_k, a_{k+1}</math>, it follows that we have strictly increased the right-hand side of the equation, i.e., | ||
− | <cmath> \sum_{1 \le i<j \le n} x_i x_j (x_i^2 + x_j^2) < sum_{1 \le i<j \le n} y_i y_j (y_i^2 + y_j^2) . </cmath> | + | <cmath> \sum_{1 \le i<j \le n} x_i x_j (x_i^2 + x_j^2) < \sum_{1 \le i<j \le n} y_i y_j (y_i^2 + y_j^2) . </cmath> |
By inductive hypothesis, | By inductive hypothesis, | ||
− | <cmath> \sum_{1\le i<j \le n} y_i y_j (y_i^2 +y_j^2) \le \left( \sum_{i=1}^n y_i right)^4 / 8 , </cmath> | + | <cmath> \sum_{1\le i<j \le n} y_i y_j (y_i^2 +y_j^2) \le \left( \sum_{i=1}^n y_i \right)^4 / 8 , </cmath> |
and by our choice of <math>y_i</math>, | and by our choice of <math>y_i</math>, | ||
<cmath> \left( \sum_{i=1}^n y_i \right)^4/8 = \left( \sum_{i=1}^n x_i \right)^4/8 .</cmath> | <cmath> \left( \sum_{i=1}^n y_i \right)^4/8 = \left( \sum_{i=1}^n x_i \right)^4/8 .</cmath> |
Revision as of 11:49, 30 December 2007
Problem
(Marcin Kuczma, Poland) Let be a fixed integer.
- (a) Find the least constant such that for all nonnegative real numbers ,
- (b) Determine when equality occurs for this value of .
Solution
The answer is , and equality holds exactly when two of the are equal to each other and all the other are zero. We prove this by induction on the number of nonzero .
First, suppose that at most two of the , say and , are nonzero. Then the left-hand side of the desired inequality becomes and the right-hand side becomes . By AM-GM, with equality exactly when , as desired.
Now, suppose that our statement holds when at most of the are equal to zero. Suppose now that of the are equal to zero, for . Without loss of generality, let these be . We define and for convenience, we will denote . We wish to show that by replacing the with the , we increase the left-hand side of the desired inequality without changing the right-hand side; and then to use the inductive hypothesis.
We note that If we replace with , then becomes but none of the other terms change. Since , it follows that we have strictly increased the right-hand side of the equation, i.e., By inductive hypothesis, and by our choice of , Hence the problem's inequality holds by induction, and is strict when there are more than two nonzero , as desired.
Alternate solutions are always welcome. If you have a different, elegant solution to this problem, please add it to this page.